Probability to win a series of three games, with a given advantage for the home team

The name of the pictureThe name of the pictureThe name of the pictureClash Royale CLAN TAG#URR8PPP











up vote
2
down vote

favorite













In a game with two equal teams, the home team wins any game with probability $p>frac12$. In a best of three playoff series, a team with a home advantage has play at home, followed by a game away, followed by a home play if necessary. The series is over as soon as one team wins two games.



What is $P[T]$, the probability that the team with home advantage wins the series? Prove that $P[T]>p$ for all $p>frac12$.




In the first sentence, I think there's a typo. I think it meant to say $p=frac12$ (I don't know how you can get a number as your answer if its not equal to $frac12$)



I drew a tree diagram as shown below, and calculated the probability that Team A wins.





$.5^2 + .5^3 + .5^3 = .5$







share|cite|improve this question





















  • There's no typo. You are using $P=frac12$ but $P>frac12$. This is a binomial distribution where some of the outcomes never happen (If A wins all it's games, the series is shorter).
    – David Diaz
    Jul 20 at 16:08







  • 1




    I think you are to calculate an answer in terms of $P$, not a numerical answer. Your tree is a good way to start.
    – Ethan Bolker
    Jul 20 at 16:11










  • if p>1/2, wouldn't P(T) be less than that?
    – David
    Jul 20 at 16:14










  • It's often easier to imagine all possible sequences of three games (even though the winner may well be decided before the third game is played). The ultimate winner in all cases is the one that wins at least two of the three. Doing it this way lets you just use the properties of the binomial distribution.
    – lulu
    Jul 20 at 16:18











  • @David You're actually right. The sense of the last inequality in the question should be inverted (see my answer for working).
    – Parcly Taxel
    Jul 20 at 16:55














up vote
2
down vote

favorite













In a game with two equal teams, the home team wins any game with probability $p>frac12$. In a best of three playoff series, a team with a home advantage has play at home, followed by a game away, followed by a home play if necessary. The series is over as soon as one team wins two games.



What is $P[T]$, the probability that the team with home advantage wins the series? Prove that $P[T]>p$ for all $p>frac12$.




In the first sentence, I think there's a typo. I think it meant to say $p=frac12$ (I don't know how you can get a number as your answer if its not equal to $frac12$)



I drew a tree diagram as shown below, and calculated the probability that Team A wins.





$.5^2 + .5^3 + .5^3 = .5$







share|cite|improve this question





















  • There's no typo. You are using $P=frac12$ but $P>frac12$. This is a binomial distribution where some of the outcomes never happen (If A wins all it's games, the series is shorter).
    – David Diaz
    Jul 20 at 16:08







  • 1




    I think you are to calculate an answer in terms of $P$, not a numerical answer. Your tree is a good way to start.
    – Ethan Bolker
    Jul 20 at 16:11










  • if p>1/2, wouldn't P(T) be less than that?
    – David
    Jul 20 at 16:14










  • It's often easier to imagine all possible sequences of three games (even though the winner may well be decided before the third game is played). The ultimate winner in all cases is the one that wins at least two of the three. Doing it this way lets you just use the properties of the binomial distribution.
    – lulu
    Jul 20 at 16:18











  • @David You're actually right. The sense of the last inequality in the question should be inverted (see my answer for working).
    – Parcly Taxel
    Jul 20 at 16:55












up vote
2
down vote

favorite









up vote
2
down vote

favorite












In a game with two equal teams, the home team wins any game with probability $p>frac12$. In a best of three playoff series, a team with a home advantage has play at home, followed by a game away, followed by a home play if necessary. The series is over as soon as one team wins two games.



What is $P[T]$, the probability that the team with home advantage wins the series? Prove that $P[T]>p$ for all $p>frac12$.




In the first sentence, I think there's a typo. I think it meant to say $p=frac12$ (I don't know how you can get a number as your answer if its not equal to $frac12$)



I drew a tree diagram as shown below, and calculated the probability that Team A wins.





$.5^2 + .5^3 + .5^3 = .5$







share|cite|improve this question














In a game with two equal teams, the home team wins any game with probability $p>frac12$. In a best of three playoff series, a team with a home advantage has play at home, followed by a game away, followed by a home play if necessary. The series is over as soon as one team wins two games.



What is $P[T]$, the probability that the team with home advantage wins the series? Prove that $P[T]>p$ for all $p>frac12$.




In the first sentence, I think there's a typo. I think it meant to say $p=frac12$ (I don't know how you can get a number as your answer if its not equal to $frac12$)



I drew a tree diagram as shown below, and calculated the probability that Team A wins.





$.5^2 + .5^3 + .5^3 = .5$









share|cite|improve this question












share|cite|improve this question




share|cite|improve this question








edited Jul 20 at 16:32









Did

242k23208443




242k23208443









asked Jul 20 at 16:05









David

603




603











  • There's no typo. You are using $P=frac12$ but $P>frac12$. This is a binomial distribution where some of the outcomes never happen (If A wins all it's games, the series is shorter).
    – David Diaz
    Jul 20 at 16:08







  • 1




    I think you are to calculate an answer in terms of $P$, not a numerical answer. Your tree is a good way to start.
    – Ethan Bolker
    Jul 20 at 16:11










  • if p>1/2, wouldn't P(T) be less than that?
    – David
    Jul 20 at 16:14










  • It's often easier to imagine all possible sequences of three games (even though the winner may well be decided before the third game is played). The ultimate winner in all cases is the one that wins at least two of the three. Doing it this way lets you just use the properties of the binomial distribution.
    – lulu
    Jul 20 at 16:18











  • @David You're actually right. The sense of the last inequality in the question should be inverted (see my answer for working).
    – Parcly Taxel
    Jul 20 at 16:55
















  • There's no typo. You are using $P=frac12$ but $P>frac12$. This is a binomial distribution where some of the outcomes never happen (If A wins all it's games, the series is shorter).
    – David Diaz
    Jul 20 at 16:08







  • 1




    I think you are to calculate an answer in terms of $P$, not a numerical answer. Your tree is a good way to start.
    – Ethan Bolker
    Jul 20 at 16:11










  • if p>1/2, wouldn't P(T) be less than that?
    – David
    Jul 20 at 16:14










  • It's often easier to imagine all possible sequences of three games (even though the winner may well be decided before the third game is played). The ultimate winner in all cases is the one that wins at least two of the three. Doing it this way lets you just use the properties of the binomial distribution.
    – lulu
    Jul 20 at 16:18











  • @David You're actually right. The sense of the last inequality in the question should be inverted (see my answer for working).
    – Parcly Taxel
    Jul 20 at 16:55















There's no typo. You are using $P=frac12$ but $P>frac12$. This is a binomial distribution where some of the outcomes never happen (If A wins all it's games, the series is shorter).
– David Diaz
Jul 20 at 16:08





There's no typo. You are using $P=frac12$ but $P>frac12$. This is a binomial distribution where some of the outcomes never happen (If A wins all it's games, the series is shorter).
– David Diaz
Jul 20 at 16:08





1




1




I think you are to calculate an answer in terms of $P$, not a numerical answer. Your tree is a good way to start.
– Ethan Bolker
Jul 20 at 16:11




I think you are to calculate an answer in terms of $P$, not a numerical answer. Your tree is a good way to start.
– Ethan Bolker
Jul 20 at 16:11












if p>1/2, wouldn't P(T) be less than that?
– David
Jul 20 at 16:14




if p>1/2, wouldn't P(T) be less than that?
– David
Jul 20 at 16:14












It's often easier to imagine all possible sequences of three games (even though the winner may well be decided before the third game is played). The ultimate winner in all cases is the one that wins at least two of the three. Doing it this way lets you just use the properties of the binomial distribution.
– lulu
Jul 20 at 16:18





It's often easier to imagine all possible sequences of three games (even though the winner may well be decided before the third game is played). The ultimate winner in all cases is the one that wins at least two of the three. Doing it this way lets you just use the properties of the binomial distribution.
– lulu
Jul 20 at 16:18













@David You're actually right. The sense of the last inequality in the question should be inverted (see my answer for working).
– Parcly Taxel
Jul 20 at 16:55




@David You're actually right. The sense of the last inequality in the question should be inverted (see my answer for working).
– Parcly Taxel
Jul 20 at 16:55










2 Answers
2






active

oldest

votes

















up vote
3
down vote













This is not a typo. $p$ is a variable and you are supposed to include it in your answer.



There are three sequences of results leading to the home advantage team winning, as you found out: WW ($p(1-p)$), WLW ($p^3$) and LWW ($(1-p)^2p$). Thus
$$P[T]=p(1-p)+p(1-p)^2+p^3=(p-p^2)(2-p)+p^3=2p-3p^2+2p^3=p(2-3p+2p^2)$$
and if $p=frac12+d$ with $d>0$
$$P[T]=pleft(2-3left(frac12+dright)+2left(frac12+dright)^2right)$$
$$=pleft(2d^2-d+1right)$$
$$=pleft(1+d(2d-1)right)<p$$
where the last inequality is because $2d-1<0$ for $0<d<frac12$. Indeed, the inequality is reversed compared to what is printed: conversely, $P[T]>p$ when $p<frac12$.






share|cite|improve this answer























  • So d is between 0 and 1/2, and P(T) is less than, not greater than p, because if you multiply any p by what you get in this equation - p(2d^2−d+1) - you get something smaller than p?
    – David
    Jul 20 at 17:49










  • @David yes. And that is the real typo in the question.
    – Parcly Taxel
    Jul 20 at 17:50

















up vote
1
down vote













I'll add an answer to point out that you can approach the analysis somewhat differently by extending the series to three games even when the same team wins the first two. This is permissible because the third game doesn't change the outcome in those cases; that is, after all, why they then don't usually play the third game. We'll have them play it just to simplify things a bit.



Then the team with home advantage wins its two home games with probability $p^3$ (the home team wins all three games) plus $2p(1-p)^2$ (it splits its home games and wins away). It wins all three games with probability $p^2(1-p)$, for a total of



$$
P(texthome team wins series) = 2p-3p^2+2p^3
$$



To see that this is actually less than $p$, we write



beginalign
2p-3p^2+2p^3-p & = p(2p^3-3p+1) \
& = p(2p-1)(p-1)
endalign



This shows that for the interval $0 leq p leq 1$, the left-hand side is positive when $0 < p < 1/2$, and negative when $1/2 < p < 1$. The latter is our situation, and since $(2p-3p^2+2p^3)-p > 0$, it must be the case that $P(texthome team wins the series) = 2p-3p^2+2p^3 > p$.






share|cite|improve this answer























  • Oops, totally misread the rules. Thanks, I'll delete and edit! :-o
    – Brian Tung
    Jul 20 at 17:06










  • Thanks @littleO for pointing out my misinterpretation (I read too quickly, is what really happened). Unfortunately, my original approach won't work, and what remains is not as simple as I'd hoped, but I'll leave the answer here just because it's a little different from Parcly Taxel's, and maybe the difference will be helpful.
    – Brian Tung
    Jul 20 at 17:19










Your Answer




StackExchange.ifUsing("editor", function ()
return StackExchange.using("mathjaxEditing", function ()
StackExchange.MarkdownEditor.creationCallbacks.add(function (editor, postfix)
StackExchange.mathjaxEditing.prepareWmdForMathJax(editor, postfix, [["$", "$"], ["\\(","\\)"]]);
);
);
, "mathjax-editing");

StackExchange.ready(function()
var channelOptions =
tags: "".split(" "),
id: "69"
;
initTagRenderer("".split(" "), "".split(" "), channelOptions);

StackExchange.using("externalEditor", function()
// Have to fire editor after snippets, if snippets enabled
if (StackExchange.settings.snippets.snippetsEnabled)
StackExchange.using("snippets", function()
createEditor();
);

else
createEditor();

);

function createEditor()
StackExchange.prepareEditor(
heartbeatType: 'answer',
convertImagesToLinks: true,
noModals: false,
showLowRepImageUploadWarning: true,
reputationToPostImages: 10,
bindNavPrevention: true,
postfix: "",
noCode: true, onDemand: true,
discardSelector: ".discard-answer"
,immediatelyShowMarkdownHelp:true
);



);








 

draft saved


draft discarded


















StackExchange.ready(
function ()
StackExchange.openid.initPostLogin('.new-post-login', 'https%3a%2f%2fmath.stackexchange.com%2fquestions%2f2857796%2fprobability-to-win-a-series-of-three-games-with-a-given-advantage-for-the-home%23new-answer', 'question_page');

);

Post as a guest






























2 Answers
2






active

oldest

votes








2 Answers
2






active

oldest

votes









active

oldest

votes






active

oldest

votes








up vote
3
down vote













This is not a typo. $p$ is a variable and you are supposed to include it in your answer.



There are three sequences of results leading to the home advantage team winning, as you found out: WW ($p(1-p)$), WLW ($p^3$) and LWW ($(1-p)^2p$). Thus
$$P[T]=p(1-p)+p(1-p)^2+p^3=(p-p^2)(2-p)+p^3=2p-3p^2+2p^3=p(2-3p+2p^2)$$
and if $p=frac12+d$ with $d>0$
$$P[T]=pleft(2-3left(frac12+dright)+2left(frac12+dright)^2right)$$
$$=pleft(2d^2-d+1right)$$
$$=pleft(1+d(2d-1)right)<p$$
where the last inequality is because $2d-1<0$ for $0<d<frac12$. Indeed, the inequality is reversed compared to what is printed: conversely, $P[T]>p$ when $p<frac12$.






share|cite|improve this answer























  • So d is between 0 and 1/2, and P(T) is less than, not greater than p, because if you multiply any p by what you get in this equation - p(2d^2−d+1) - you get something smaller than p?
    – David
    Jul 20 at 17:49










  • @David yes. And that is the real typo in the question.
    – Parcly Taxel
    Jul 20 at 17:50














up vote
3
down vote













This is not a typo. $p$ is a variable and you are supposed to include it in your answer.



There are three sequences of results leading to the home advantage team winning, as you found out: WW ($p(1-p)$), WLW ($p^3$) and LWW ($(1-p)^2p$). Thus
$$P[T]=p(1-p)+p(1-p)^2+p^3=(p-p^2)(2-p)+p^3=2p-3p^2+2p^3=p(2-3p+2p^2)$$
and if $p=frac12+d$ with $d>0$
$$P[T]=pleft(2-3left(frac12+dright)+2left(frac12+dright)^2right)$$
$$=pleft(2d^2-d+1right)$$
$$=pleft(1+d(2d-1)right)<p$$
where the last inequality is because $2d-1<0$ for $0<d<frac12$. Indeed, the inequality is reversed compared to what is printed: conversely, $P[T]>p$ when $p<frac12$.






share|cite|improve this answer























  • So d is between 0 and 1/2, and P(T) is less than, not greater than p, because if you multiply any p by what you get in this equation - p(2d^2−d+1) - you get something smaller than p?
    – David
    Jul 20 at 17:49










  • @David yes. And that is the real typo in the question.
    – Parcly Taxel
    Jul 20 at 17:50












up vote
3
down vote










up vote
3
down vote









This is not a typo. $p$ is a variable and you are supposed to include it in your answer.



There are three sequences of results leading to the home advantage team winning, as you found out: WW ($p(1-p)$), WLW ($p^3$) and LWW ($(1-p)^2p$). Thus
$$P[T]=p(1-p)+p(1-p)^2+p^3=(p-p^2)(2-p)+p^3=2p-3p^2+2p^3=p(2-3p+2p^2)$$
and if $p=frac12+d$ with $d>0$
$$P[T]=pleft(2-3left(frac12+dright)+2left(frac12+dright)^2right)$$
$$=pleft(2d^2-d+1right)$$
$$=pleft(1+d(2d-1)right)<p$$
where the last inequality is because $2d-1<0$ for $0<d<frac12$. Indeed, the inequality is reversed compared to what is printed: conversely, $P[T]>p$ when $p<frac12$.






share|cite|improve this answer















This is not a typo. $p$ is a variable and you are supposed to include it in your answer.



There are three sequences of results leading to the home advantage team winning, as you found out: WW ($p(1-p)$), WLW ($p^3$) and LWW ($(1-p)^2p$). Thus
$$P[T]=p(1-p)+p(1-p)^2+p^3=(p-p^2)(2-p)+p^3=2p-3p^2+2p^3=p(2-3p+2p^2)$$
and if $p=frac12+d$ with $d>0$
$$P[T]=pleft(2-3left(frac12+dright)+2left(frac12+dright)^2right)$$
$$=pleft(2d^2-d+1right)$$
$$=pleft(1+d(2d-1)right)<p$$
where the last inequality is because $2d-1<0$ for $0<d<frac12$. Indeed, the inequality is reversed compared to what is printed: conversely, $P[T]>p$ when $p<frac12$.







share|cite|improve this answer















share|cite|improve this answer



share|cite|improve this answer








edited Jul 20 at 16:53


























answered Jul 20 at 16:27









Parcly Taxel

33.6k136588




33.6k136588











  • So d is between 0 and 1/2, and P(T) is less than, not greater than p, because if you multiply any p by what you get in this equation - p(2d^2−d+1) - you get something smaller than p?
    – David
    Jul 20 at 17:49










  • @David yes. And that is the real typo in the question.
    – Parcly Taxel
    Jul 20 at 17:50
















  • So d is between 0 and 1/2, and P(T) is less than, not greater than p, because if you multiply any p by what you get in this equation - p(2d^2−d+1) - you get something smaller than p?
    – David
    Jul 20 at 17:49










  • @David yes. And that is the real typo in the question.
    – Parcly Taxel
    Jul 20 at 17:50















So d is between 0 and 1/2, and P(T) is less than, not greater than p, because if you multiply any p by what you get in this equation - p(2d^2−d+1) - you get something smaller than p?
– David
Jul 20 at 17:49




So d is between 0 and 1/2, and P(T) is less than, not greater than p, because if you multiply any p by what you get in this equation - p(2d^2−d+1) - you get something smaller than p?
– David
Jul 20 at 17:49












@David yes. And that is the real typo in the question.
– Parcly Taxel
Jul 20 at 17:50




@David yes. And that is the real typo in the question.
– Parcly Taxel
Jul 20 at 17:50










up vote
1
down vote













I'll add an answer to point out that you can approach the analysis somewhat differently by extending the series to three games even when the same team wins the first two. This is permissible because the third game doesn't change the outcome in those cases; that is, after all, why they then don't usually play the third game. We'll have them play it just to simplify things a bit.



Then the team with home advantage wins its two home games with probability $p^3$ (the home team wins all three games) plus $2p(1-p)^2$ (it splits its home games and wins away). It wins all three games with probability $p^2(1-p)$, for a total of



$$
P(texthome team wins series) = 2p-3p^2+2p^3
$$



To see that this is actually less than $p$, we write



beginalign
2p-3p^2+2p^3-p & = p(2p^3-3p+1) \
& = p(2p-1)(p-1)
endalign



This shows that for the interval $0 leq p leq 1$, the left-hand side is positive when $0 < p < 1/2$, and negative when $1/2 < p < 1$. The latter is our situation, and since $(2p-3p^2+2p^3)-p > 0$, it must be the case that $P(texthome team wins the series) = 2p-3p^2+2p^3 > p$.






share|cite|improve this answer























  • Oops, totally misread the rules. Thanks, I'll delete and edit! :-o
    – Brian Tung
    Jul 20 at 17:06










  • Thanks @littleO for pointing out my misinterpretation (I read too quickly, is what really happened). Unfortunately, my original approach won't work, and what remains is not as simple as I'd hoped, but I'll leave the answer here just because it's a little different from Parcly Taxel's, and maybe the difference will be helpful.
    – Brian Tung
    Jul 20 at 17:19














up vote
1
down vote













I'll add an answer to point out that you can approach the analysis somewhat differently by extending the series to three games even when the same team wins the first two. This is permissible because the third game doesn't change the outcome in those cases; that is, after all, why they then don't usually play the third game. We'll have them play it just to simplify things a bit.



Then the team with home advantage wins its two home games with probability $p^3$ (the home team wins all three games) plus $2p(1-p)^2$ (it splits its home games and wins away). It wins all three games with probability $p^2(1-p)$, for a total of



$$
P(texthome team wins series) = 2p-3p^2+2p^3
$$



To see that this is actually less than $p$, we write



beginalign
2p-3p^2+2p^3-p & = p(2p^3-3p+1) \
& = p(2p-1)(p-1)
endalign



This shows that for the interval $0 leq p leq 1$, the left-hand side is positive when $0 < p < 1/2$, and negative when $1/2 < p < 1$. The latter is our situation, and since $(2p-3p^2+2p^3)-p > 0$, it must be the case that $P(texthome team wins the series) = 2p-3p^2+2p^3 > p$.






share|cite|improve this answer























  • Oops, totally misread the rules. Thanks, I'll delete and edit! :-o
    – Brian Tung
    Jul 20 at 17:06










  • Thanks @littleO for pointing out my misinterpretation (I read too quickly, is what really happened). Unfortunately, my original approach won't work, and what remains is not as simple as I'd hoped, but I'll leave the answer here just because it's a little different from Parcly Taxel's, and maybe the difference will be helpful.
    – Brian Tung
    Jul 20 at 17:19












up vote
1
down vote










up vote
1
down vote









I'll add an answer to point out that you can approach the analysis somewhat differently by extending the series to three games even when the same team wins the first two. This is permissible because the third game doesn't change the outcome in those cases; that is, after all, why they then don't usually play the third game. We'll have them play it just to simplify things a bit.



Then the team with home advantage wins its two home games with probability $p^3$ (the home team wins all three games) plus $2p(1-p)^2$ (it splits its home games and wins away). It wins all three games with probability $p^2(1-p)$, for a total of



$$
P(texthome team wins series) = 2p-3p^2+2p^3
$$



To see that this is actually less than $p$, we write



beginalign
2p-3p^2+2p^3-p & = p(2p^3-3p+1) \
& = p(2p-1)(p-1)
endalign



This shows that for the interval $0 leq p leq 1$, the left-hand side is positive when $0 < p < 1/2$, and negative when $1/2 < p < 1$. The latter is our situation, and since $(2p-3p^2+2p^3)-p > 0$, it must be the case that $P(texthome team wins the series) = 2p-3p^2+2p^3 > p$.






share|cite|improve this answer















I'll add an answer to point out that you can approach the analysis somewhat differently by extending the series to three games even when the same team wins the first two. This is permissible because the third game doesn't change the outcome in those cases; that is, after all, why they then don't usually play the third game. We'll have them play it just to simplify things a bit.



Then the team with home advantage wins its two home games with probability $p^3$ (the home team wins all three games) plus $2p(1-p)^2$ (it splits its home games and wins away). It wins all three games with probability $p^2(1-p)$, for a total of



$$
P(texthome team wins series) = 2p-3p^2+2p^3
$$



To see that this is actually less than $p$, we write



beginalign
2p-3p^2+2p^3-p & = p(2p^3-3p+1) \
& = p(2p-1)(p-1)
endalign



This shows that for the interval $0 leq p leq 1$, the left-hand side is positive when $0 < p < 1/2$, and negative when $1/2 < p < 1$. The latter is our situation, and since $(2p-3p^2+2p^3)-p > 0$, it must be the case that $P(texthome team wins the series) = 2p-3p^2+2p^3 > p$.







share|cite|improve this answer















share|cite|improve this answer



share|cite|improve this answer








edited Jul 20 at 17:18


























answered Jul 20 at 16:59









Brian Tung

25.2k32453




25.2k32453











  • Oops, totally misread the rules. Thanks, I'll delete and edit! :-o
    – Brian Tung
    Jul 20 at 17:06










  • Thanks @littleO for pointing out my misinterpretation (I read too quickly, is what really happened). Unfortunately, my original approach won't work, and what remains is not as simple as I'd hoped, but I'll leave the answer here just because it's a little different from Parcly Taxel's, and maybe the difference will be helpful.
    – Brian Tung
    Jul 20 at 17:19
















  • Oops, totally misread the rules. Thanks, I'll delete and edit! :-o
    – Brian Tung
    Jul 20 at 17:06










  • Thanks @littleO for pointing out my misinterpretation (I read too quickly, is what really happened). Unfortunately, my original approach won't work, and what remains is not as simple as I'd hoped, but I'll leave the answer here just because it's a little different from Parcly Taxel's, and maybe the difference will be helpful.
    – Brian Tung
    Jul 20 at 17:19















Oops, totally misread the rules. Thanks, I'll delete and edit! :-o
– Brian Tung
Jul 20 at 17:06




Oops, totally misread the rules. Thanks, I'll delete and edit! :-o
– Brian Tung
Jul 20 at 17:06












Thanks @littleO for pointing out my misinterpretation (I read too quickly, is what really happened). Unfortunately, my original approach won't work, and what remains is not as simple as I'd hoped, but I'll leave the answer here just because it's a little different from Parcly Taxel's, and maybe the difference will be helpful.
– Brian Tung
Jul 20 at 17:19




Thanks @littleO for pointing out my misinterpretation (I read too quickly, is what really happened). Unfortunately, my original approach won't work, and what remains is not as simple as I'd hoped, but I'll leave the answer here just because it's a little different from Parcly Taxel's, and maybe the difference will be helpful.
– Brian Tung
Jul 20 at 17:19












 

draft saved


draft discarded


























 


draft saved


draft discarded














StackExchange.ready(
function ()
StackExchange.openid.initPostLogin('.new-post-login', 'https%3a%2f%2fmath.stackexchange.com%2fquestions%2f2857796%2fprobability-to-win-a-series-of-three-games-with-a-given-advantage-for-the-home%23new-answer', 'question_page');

);

Post as a guest













































































Comments

Popular posts from this blog

Color the edges and diagonals of a regular polygon

Relationship between determinant of matrix and determinant of adjoint?

What is the equation of a 3D cone with generalised tilt?